Precalculus (6th Edition)

Published by Pearson
ISBN 10: 013421742X
ISBN 13: 978-0-13421-742-0

Chapter 5 - Trigonometric Functions - 5.3 Trigonometric Functions Values and Angle Measures - 5.3 Exercises - Page 533: 108

Answer

$ 135^{\circ}$ and $315^{\circ}$.

Work Step by Step

$\cot\theta=-1$ $\tan\theta=\frac{1}{\cot\theta}=-1$ The value of $\tan \theta$ is negative, so $\theta$ may lie in either quadrant II or IV. We know that $\tan 45^{\circ}=1$. Recall: $-\tan x= \tan(180^{\circ}-x)$ and $-\tan x= \tan(360^{\circ}-x)$ $\implies -\tan 45^{\circ}=-1=\tan(180^{\circ}-45^{\circ})$ $=\tan 135^{\circ}$ $135^{\circ}$ which is in the second quadrant is one value of $\theta$. Now $-\tan 45^{\circ}=-1= \tan (360^{\circ}-45^{\circ})$ $=\tan 315^{\circ}$ $315^{\circ}$ is in the fourth quadrant. The values of $\theta$ are $ 135^{\circ}$ and $315^{\circ}$.
Update this answer!

You can help us out by revising, improving and updating this answer.

Update this answer

After you claim an answer you’ll have 24 hours to send in a draft. An editor will review the submission and either publish your submission or provide feedback.